Clockwise or counterclockwise? (linear system phase portrait)

  • Thread starter Thread starter ak416
  • Start date Start date
  • Tags Tags
    Phase System
Click For Summary
To determine the rotational direction of solutions in a linear system with a 2x2 matrix A featuring complex eigenvalues, one can analyze the signs of the derivatives of the system. Specifically, a clockwise rotation is indicated when x1' > 0 and x2' < 0 for positive x1 and x2, provided their values are not too small. Evaluating the matrix's effect on the standard basis vectors (1, 0) and (0, 1) can also clarify the rotation direction. For instance, the matrix [[0, -1], [1, 0]] results in counter-clockwise rotation, while [[0, 1], [-1, 0]] produces clockwise rotation despite having the same eigenvalues. Understanding these relationships helps in classifying the behavior of the system's solutions.
ak416
Messages
121
Reaction score
0

Homework Statement



Given a 2x2 matrix A with entries a,b,c,d (real) with complex eigenvalues I would like to know how to find out whether the solutions to the linear system are clockwise or counterclockwise. (Some kind of inequality between a,b,c,d).


Homework Equations





The Attempt at a Solution



I tried looking at the signs of each component of the derivative. It seems to me that clockwise means x1' > 0 and x2' < 0 for x1,x2 > 0 as long as x1 or x2 are not too small. Then I am not sure...
 
Physics news on Phys.org
The simplest way is to see what the matrix does to (1, 0) and (0, 1).

For example, if the problem is
\frac{dX}{dt}= \left( \begin{array}{cc}0 &amp; -1\\ 1 &amp; 0\end{array}\right)X
Then
\left( \begin{array}{cc}0 &amp; -1\\ 1 &amp; 0\end{array}\right)\left(\begin{array}{c} 1 \\ 0\end{array}\right)= \left(\begin{array}{c}0 \\ 1\end{array}\right)
That's counter-clockwise rotation. The matrix
\left(\begin{array}{cc}0 &amp; 1\\ -1 &amp; 0\end{array}\right)
has exactly the same eigenvalues but is clockwise rotation.
 
Question: A clock's minute hand has length 4 and its hour hand has length 3. What is the distance between the tips at the moment when it is increasing most rapidly?(Putnam Exam Question) Answer: Making assumption that both the hands moves at constant angular velocities, the answer is ## \sqrt{7} .## But don't you think this assumption is somewhat doubtful and wrong?

Similar threads

  • · Replies 1 ·
Replies
1
Views
2K
  • · Replies 24 ·
Replies
24
Views
3K
Replies
2
Views
2K
Replies
14
Views
2K
  • · Replies 3 ·
Replies
3
Views
6K
Replies
6
Views
2K
  • · Replies 4 ·
Replies
4
Views
2K
Replies
2
Views
2K
  • · Replies 8 ·
Replies
8
Views
3K
  • · Replies 1 ·
Replies
1
Views
1K